Your-Doctor
Multiple Choice Questions (MCQ)



Free Palestine
Quiz Categories Click to expand

Category: Prometric--->Neurology
Page: 26

Question 126# Print Question

A 49-year-old man presents to the Emergency Department complaining of visual disturbance. Examination reveals a right incongruous homonymous hemianopia.

Where is the lesion most likely to be?

A. Left optic tract
B. Left optic radiation
C. Right optic tract
D. Right optic radiation
E. Optic chiasm


Question 127# Print Question

Each one of the following is associated with Friedreich's ataxia, except:

A. Increased risk of deep vein thrombosis
B. Optic atrophy
C. Cardiomyopathy
D. Nystagmus
E. High-arched palate


Question 128# Print Question

A 61-year-old woman presents with bilateral tinnitus. She reports no change in her hearing or other ear-related symptoms. Ear and cranial nerve examination is unremarkable.

Which medication is she most likely to have recently started?

A. Ciprofloxacin
B. Nifedipine
C. Repaglinide
D. Quinine
E. Bendroflumethiazide


Question 129# Print Question

What is the most common clinical pattern seen in motor neuron disease?

A. Progressive muscular atrophy
B. Progressive bulbar palsy
C. Spinocerebellar ataxia
D. Relapsing-remitting
E. Amyotrophic lateral sclerosis


Question 130# Print Question

Each one of the following is associated with ataxic telangiectasia, except:

A. Telangiectasia
B. Cerebellar ataxia
C. Autosomal dominant inheritance
D. Recurrent chest infections
E. Increased risk of malignancy




Category: Prometric--->Neurology
Page: 26 of 41